2018 AMC 10A Problems/Problem 1

Revision as of 14:57, 8 February 2018 by Ilike3.1415926 (talk | contribs) (Created page with "What is the value of <cmath>\left(\left((2+1)^{-1}+1\right)^{-1}+1\right)^{-1}+1?</cmath><math>\textbf{(A) } \frac58 \qquad \textbf{(B) }\frac{11}7 \qquad \textbf{(C) } \frac8...")
(diff) ← Older revision | Latest revision (diff) | Newer revision → (diff)

What is the value of \[\left(\left((2+1)^{-1}+1\right)^{-1}+1\right)^{-1}+1?\]$\textbf{(A) } \frac58 \qquad \textbf{(B) }\frac{11}7 \qquad \textbf{(C) } \frac85 \qquad \textbf{(D) } \frac{18}{11} \qquad \textbf{(E) } \frac{15}8$

Solution

Evaluating the expressions starting with the innermost one, we get that the answer is $\boxed{\textbf{(B)}\ \frac{11}{7}}$